endocrine and reproductive system Flashcards

1
Q
  1. Which receptor controls nitric oxide (NO) release to
    cause vasodilation during penile erection?
    A) Leptin receptor
    B) Angiotensin AT1 receptor
    C) Endothelin ETA receptor
    D) Muscarinic receptor
A

D) Muscarinic receptor

How well did you know this?
1
Not at all
2
3
4
5
Perfectly
2
Q
  1. Which statement about antidiuretic hormone (ADH)
    is true?
    A) It is synthesized in the posterior pituitary gland
    B) It increases salt and water reabsorption in the collecting tubules and ducts
    C) It stimulates thirst
    D) It has opposite effects on urine and plasma osmolality
A

D) It has opposite effects on urine and plasma osmolality

How well did you know this?
1
Not at all
2
3
4
5
Perfectly
3
Q
  1. After menopause, hormone replacement therapy with
    estrogen-like compounds is effective in preventing the
    progression of osteoporosis. What is the mechanism of
    their protective effect?
    A) They stimulate the activity of osteoblasts
    B) They increase absorption of calcium from the gastrointestinal tract
    C) They stimulate calcium reabsorption by the renal
    tubules
    D) They stimulate parathyroid hormone (PTH) secretion by the parathyroid glan
A

A) They stimulate the activity of osteoblasts

How well did you know this?
1
Not at all
2
3
4
5
Perfectly
4
Q
  1. A patient has nephrogenic diabetes insipidus. Of the
    following options, which outcome would be expected
    or which intervention would be suggested?
    A) Expected outcome: decreased plasma sodium concentration
    B) Expected outcome: increased secretion of ADH
    from the supraoptic and paraventricular nuclei
    C) Expected outcome: high urine osmolality
    D) Suggested intervention: water restriction
    E) Suggested intervention: ADH antagonists (vaptans)
A

B) Expected outcome: increased secretion of ADH

from the supraoptic and paraventricular nuclei

How well did you know this?
1
Not at all
2
3
4
5
Perfectly
5
Q
  1. Within minutes after a normal delivery, flow through
    the foramen ovale decreases dramatically. What is the
    cause of this change?
    A) Increased formation of prostaglandin E2 (PGE2) in
    the endocardium
    B) Increased rate of flow through the pulmonary
    artery
    C) Increased left atrial pressure
    D) Increased right atrial pressure
    E) Increased partial pressure of oxygen (Po2)
A

C) Increased left atrial pressure

How well did you know this?
1
Not at all
2
3
4
5
Perfectly
6
Q
  1. Which hormones antagonize the effect of NO and cause the penis to become flaccid after orgasm?
    A) Endothelin and norepinephrine
    B) Estrogen and progesterone
    C) Luteinizing hormone (LH) and follicle-stimulating
    hormone (FSH)
    D) Progesterone and LH
A

A) Endothelin and norepinephrine

How well did you know this?
1
Not at all
2
3
4
5
Perfectly
7
Q
10. Thecal cells in the follicle are not able to produce what 
sex steroid?
A) Estradiol
B) Testosterone
C) Progesterone
D) Dihydrotestosterone
A

A) Estradiol

How well did you know this?
1
Not at all
2
3
4
5
Perfectly
8
Q
  1. A baby is born with a penis, a scrotum with no testes,
    no vagina, and XX chromosomes. This condition is referred to as hermaphroditism. What could cause this
    abnormality?
    A) Abnormally high levels of human chorionic gonadotropin (HCG) production by the trophoblast cells
    B) The presence of a testosterone-secreting tumor in
    the mother’s right adrenal gland
    C) Abnormally high levels of LH in the maternal blood
    D) Abnormally low levels of testosterone in the maternal blood
    E) Abnormally low rates of estrogen production by
    the placenta
A

B) The presence of a testosterone-secreting tumor in

the mother’s right adrenal gland

How well did you know this?
1
Not at all
2
3
4
5
Perfectly
9
Q
  1. A young woman is given daily injections of a substance
    beginning on the sixteenth day of her normal menstrual cycle and continuing for 3 weeks. As long as the
    injections continue, she does not menstruate. The injected substance could be which of the following?
    A) Testosterone
    B) FSH
    C) An inhibitor of progesterone’s actions
    D) A PGE2 inhibitor
    E) HCG
A

E) HCG

How well did you know this?
1
Not at all
2
3
4
5
Perfectly
10
Q
13. Which of the following increases secretion of GH?
A) Senescence
B) Insulin-like growth factor-1 (IGF-1)
C) Somatostatin
D) Hypoglycemia
E) Exogenous GH administration
A

D) Hypoglycemia

How well did you know this?
1
Not at all
2
3
4
5
Perfectly
11
Q
  1. Which of the following could inhibit the initiation of
    labor?
    A) Administration of an antagonist of the actions of
    progesterone
    B) Administration of LH
    C) Administration of an antagonist of PGE2 effects
    D) Mechanically dilating and stimulating the cervix
    E) Administration of oxytocin
A

C) Administration of an antagonist of PGE2 effects

How well did you know this?
1
Not at all
2
3
4
5
Perfectly
12
Q
  1. Exposure to ultraviolet light directly facilitates which
    of the following?

A) Conversion of cholesterol to 25-hydroxycholicalciferol
B) Conversion of 25-hydroxycholicalciferol to 1,25-
dihydroxycholicalciferol
C) Transport of calcium into the extracellular fluid
D) Formation of calcium-binding protein
E) Storage of vitamin D3 in the liver

A

A) Conversion of cholesterol to 25-hydroxycholicalciferol

How well did you know this?
1
Not at all
2
3
4
5
Perfectly
13
Q
  1. Which of the following decreases the pressure in the
    pulmonary artery after birth?
    A) An increase in systemic arterial pressure
    B) Closure of ductus arteriosus
    C) An increase in left ventricular pressure
    D) A decrease in pulmonary vascular resistance
A

D) A decrease in pulmonary vascular resistance

How well did you know this?
1
Not at all
2
3
4
5
Perfectly
14
Q
17. Which of the following is both synthesized and stored 
in the hypothalamus?
A) ADH
B) Thyroid-stimulating hormone (TSH)
C) LH
D) Somatostatin
E) Somatomedin
A

D) Somatostatin

How well did you know this?
1
Not at all
2
3
4
5
Perfectly
15
Q
  1. If a radioimmunoassay is properly conducted and the
    amount of radioactive hormone bound to antibody is
    low, what would this result indicate?
    A) Plasma levels of endogenous hormone are high
    B) Plasma levels of endogenous hormone are low
    C) More antibody is needed
    D) Less radioactive hormone is needed
A

A) Plasma levels of endogenous hormone are high

How well did you know this?
1
Not at all
2
3
4
5
Perfectly
16
Q
  1. By which mechanism do LH and FSH return to baseline levels?
    A) LH surge
    B) Negative feedback on gonadotropin-releasing hormone (GnRH) by progesterone
    C) Negative feedback on GnRH by estradiol
    D) Negative feedback on GnRH from testosterone
A

C) Negative feedback on GnRH by estradiol

How well did you know this?
1
Not at all
2
3
4
5
Perfectly
17
Q
  1. Spermatogenesis is regulated by a negative feedback
    control system in which FSH stimulates the steps in
    sperm cell formation. Which negative feedback signal
    associated with sperm cell production inhibits pituitary formation of FSH?
    A) Testosterone
    B) Inhibin
    C) Estrogen
    D) LH
A

B) Inhibin

How well did you know this?
1
Not at all
2
3
4
5
Perfectly
18
Q
  1. Which of the following is true during the 12-hour
    period preceding ovulation?
    A) A surge of LH is secreted from the pituitary
    B) The surge occurs immediately after the formation
    of the corpus luteum
    C) The surge is followed immediately by a fall in the
    plasma concentration of progesterone
    D) The number of developing follicles is increasing
A

B) The surge occurs immediately after the formation

of the corpus luteum

How well did you know this?
1
Not at all
2
3
4
5
Perfectly
19
Q
  1. When do progesterone levels rise to their highest point
    during the female hormonal cycle?
    A) Between ovulation and the beginning of menstruation
    B) Immediately before ovulation
    C) When the blood concentration of LH is at its highest point
    D) When 12 primary follicles are developing to the antral stag
A

A) Between ovulation and the beginning of menstruation

How well did you know this?
1
Not at all
2
3
4
5
Perfectly
20
Q
  1. What accompanies sloughing of the endometrium during the endometrial cycle in a normal woman?
    A) An increase in progesterone
    B) The LH “surge”
    C) A decrease in both progesterone and estrogen
    D) An increase in estradiol
A

C) A decrease in both progesterone and estrogen

How well did you know this?
1
Not at all
2
3
4
5
Perfectly
21
Q
  1. Some cells secrete chemicals into the extracellular fluid
    that act on cells in the same tissue. Which of the following refers to this type of regulation?
    A) Neural
    B) Endocrine
    C) Neuroendocrine
    D) Paracrine
    E) Autocrine
A

D) Paracrine

How well did you know this?
1
Not at all
2
3
4
5
Perfectly
22
Q
  1. Which of the following pairs is an example of the type
    of regulation referred to in Question 24?
    A) Somatostatin—GH secretion
    B) Somatostatin—insulin secretion
    C) Dopamine—prolactin secretion
    D) Norepinephrine—corticotropin-releasing hormone
    (CRH) secretion
    E) CRH—adrenocorticotropic hormone (ACTH)
    secretion
A

B) Somatostatin—insulin secretion

How well did you know this?
1
Not at all
2
3
4
5
Perfectly
23
Q
  1. A professional athlete in her mid-20s has not had a
    menstrual cycle for 5 years, although a bone density
    scan revealed normal skeletal mineralization. Which
    fact may explain these observations?
    A) She consumes a high-carbohydrate diet
    B) Her grandmother sustained a hip fracture at age
    79 years
    C) Her blood pressure is higher than normal
    D) Her plasma estrogen concentration is very low
    E) She has been taking anabolic steroid supplements
    for 5 years
A

E) She has been taking anabolic steroid supplements

for 5 years

How well did you know this?
1
Not at all
2
3
4
5
Perfectly
24
Q
27. What is the nongenomic effect of testosterone on vascular smooth muscle?
A) Vasodilation
B) Vasoconstriction
C) Increase in prostaglandins
D) Increase in estrogen receptors
A

A) Vasodilation

How well did you know this?
1
Not at all
2
3
4
5
Perfectly
25
Q
28. In the circulatory system of a fetus, which of the following is greater before birth than after birth?
A) Arterial Po2
B) Right atrial pressure
C) Aortic pressure
D) Left ventricular pressure
A

B) Right atrial pressure

How well did you know this?
1
Not at all
2
3
4
5
Perfectly
26
Q
32. Which enzyme in the cytochrome P450 steroid synthesis cascade is directly responsible for estradiol 
synthesis?
A) 17-beta-hydroxysteroid dehydrogenase
B) 5-alpha reductase
C) Aromatase
D) Side chain cleavage enzyme
A

C) Aromatase

How well did you know this?
1
Not at all
2
3
4
5
Perfectly
27
Q
33. Which of the following is greater after birth than before birth?
A) Flow through the foramen ovale
B) Pressure in the right atrium
C) Flow through the ductus arteriosus
D) Aortic pressure
A

D) Aortic pressure

How well did you know this?
1
Not at all
2
3
4
5
Perfectly
28
Q
  1. PTH does what directly?
    A) Controls the rate of 25-hydroxycholicalciferol
    formation
    B) Controls the rate of calcium transport in the mucosa of the small intestine
    C) Controls the rate of formation of calcium-binding
    protein
    D) Controls the rate of formation of 1,25-dihydroxycholicalciferol
    E) Stimulates renal tubular phosphate reabsorption
A

D) Controls the rate of formation of 1,25-dihydroxycholicalciferol

How well did you know this?
1
Not at all
2
3
4
5
Perfectly
29
Q
35. Which substances are most likely to produce the greatest increase in insulin secretion?
A) Amino acids
B) Amino acids and glucose
C) Amino acids and somatostatin
D) Glucose and somatostatin
A

B) Amino acids and glucose

How well did you know this?
1
Not at all
2
3
4
5
Perfectly
30
Q
  1. For male differentiation to occur during embryonic
    development, testosterone must be secreted from the
    testes. What stimulates the secretion of testosterone
    during embryonic development?
    A) LH from the maternal pituitary gland
    B) HCG
    C) Inhibin from the corpus luteum
    D) GnRH from the embryo’s hypothalamus
A

B) HCG

How well did you know this?
1
Not at all
2
3
4
5
Perfectly
31
Q
  1. A patient has an elevated plasma thyroxine (T4) concentration, a low plasma TSH concentration, and her
    thyroid gland is smaller than normal. What is the most
    likely explanation for these findings?
    A) A lesion in the anterior pituitary that prevents TSH
    secretion
    B) The patient is taking propylthiouracil
    C) The patient is taking thyroid extract
    D) The patient is consuming large amounts of iodine
    E) Graves’ disease
A

C) The patient is taking thyroid extract

How well did you know this?
1
Not at all
2
3
4
5
Perfectly
32
Q
  1. Extracellular ionic calcium activity will be decreased
    within 1 minute by which of the following?
    A) An increase in extracellular phosphate ion activity
    B) An increase in extracellular pH
    C) A decrease in extracellular partial pressure of carbon dioxide (Pco2)
    D) All the above
    E) None of the above
A

D) All the above

How well did you know this?
1
Not at all
2
3
4
5
Perfectly
33
Q
  1. As menstruation ends, estrogen levels in the blood rise
    rapidly. What is the source of the estrogen?
    A) Corpus luteum
    B) Developing follicles
    C) Endometrium
    D) Stromal cells of the ovaries
    E) Anterior pituitary gland
A

B) Developing follicles

How well did you know this?
1
Not at all
2
3
4
5
Perfectly
34
Q
  1. A 30-year-old woman reports to the clinic for a routine
    physical examination. The examination reveals she is
    pregnant. Her plasma levels of TSH are high, but her
    total thyroid hormone concentration is normal. Which
    of the following best reflects the patient’s clinical state?
    A) Graves’ disease
    B) Hashimoto’s disease
    C) A pituitary tumor secreting TSH
    D) A hypothalamic tumor secreting thyrotropinreleasing hormone (TRH)
    E) The patient is taking thyroid extract
A

B) Hashimoto’s disease

How well did you know this?
1
Not at all
2
3
4
5
Perfectly
35
Q
  1. Which anterior pituitary hormone plays a major role in
    the regulation of a nonendocrine target gland?
    A) ACTH
    B) TSH
    C) Prolactin
    D) FSH
    E) LH
A

C) Prolactin

How well did you know this?
1
Not at all
2
3
4
5
Perfectly
36
Q
  1. A female athlete who took testosterone-like steroids
    for several months stopped having normal menstrual
    cycles. What is the best explanation for this observation?
    A) Testosterone stimulates inhibin production from
    the corpus luteum
    B) Testosterone binds to receptors in the endometrium, resulting in the failure of the endometrium to
    develop during the normal cycle
    C) Testosterone binds to receptors in the anterior
    pituitary that stimulate the secretion of FSH and LH
    D) Testosterone inhibits the hypothalamic secretion of
    GnRH and the pituitary secretion of LH and FSH
A

D) Testosterone inhibits the hypothalamic secretion of

GnRH and the pituitary secretion of LH and FSH

How well did you know this?
1
Not at all
2
3
4
5
Perfectly
37
Q
  1. Which of the following decreases the resistance in the
    arteries leading to the sinuses of the penis?
    A) Stimulation of the sympathetic nerves innervating
    the arteries
    B) NO
    C) Inhibition of activity of the parasympathetic nerves
    leading to the arteries
    D) All the above
A

B) NO

How well did you know this?
1
Not at all
2
3
4
5
Perfectly
38
Q
  1. A patient has a goiter associated with high plasma levels of both TRH and TSH. Her heart rate is elevated.
    This patient most likely has which condition?
    A) An endemic goiter
    B) A hypothalamic tumor secreting large amounts of
    TRH
    C) A pituitary tumor secreting large amounts of TSH
    D) Graves’ disease
A

B) A hypothalamic tumor secreting large amounts of

TRH

How well did you know this?
1
Not at all
2
3
4
5
Perfectly
39
Q
  1. A man eats a low-carbohydrate meal that is rich in proteins containing the amino acids that stimulate insulin
    secretion. Which response accounts for the absence of
    hypoglycemia?
    A) Suppression of GH
    B) Suppression of somatomedin C secretion
    C) Stimulation of cortisol secretion
    D) Stimulation of glucagon secretion
    E) Stimulation of epinephrine secretion
A

D) Stimulation of glucagon secretion

How well did you know this?
1
Not at all
2
3
4
5
Perfectly
40
Q
  1. A 46-year-old man has “puffy” skin and is lethargic. His
    plasma TSH concentration is low and increases markedly when he is given TRH. What is the most likely
    diagnosis?
    A) Hyperthyroidism due to a thyroid tumor
    B) Hyperthyroidism due to an abnormality in the
    hypothalamus
    C) Hypothyroidism due to an abnormality in the thyroid
    D) Hypothyroidism due to an abnormality in the
    hypothalamus
    E) Hypothyroidism due to an abnormality in the pituitary
A

D) Hypothyroidism due to an abnormality in the

hypothalamus

How well did you know this?
1
Not at all
2
3
4
5
Perfectly
41
Q
  1. Negative feedback on FSH release from the anterior
    pituitary in men that results in a reduction in estradiol
    production is due to which hormone?
    A) Progesterone
    B) Estradiol
    C) Testosterone
    D) Inhibin
A

D) Inhibin

How well did you know this?
1
Not at all
2
3
4
5
Perfectly
42
Q
  1. During the first few years after menopause, FSH levels are normally extremely high. A 56-year-old woman
    completed menopause 3 years ago. However, she is
    found to have low levels of FSH in her blood. What is
    the best explanation for this finding?
    A) She has been receiving hormone replacement therapy with estrogen and progesterone since she completed menopause
    B) Her adrenal glands continue to produce estrogen
    C) Her ovaries continue to secrete estrogen
    D) She took birth control pills for 20 years before
    menopause
A

A) She has been receiving hormone replacement therapy with estrogen and progesterone since she completed menopause

How well did you know this?
1
Not at all
2
3
4
5
Perfectly
43
Q
51. Blockade of what receptors will prolong erection in the 
male?
A) Estrogen receptors
B) Cholesterol receptors
C) Muscarinic receptors
D) Phosphodiesterase-5 receptors
A

D) Phosphodiesterase-5 receptors

How well did you know this?
1
Not at all
2
3
4
5
Perfectly
44
Q
  1. Which of the following pairs of hormones and the corresponding action is incorrect?
    A) Glucagon—increased glycogenolysis in liver
    B) Glucagon—increased glycogenolysis in skeletal
    muscle
    C) Glucagon—increased gluconeogenesis
    D) Cortisol—increased gluconeogenesis
    E) Cortisol—decreased glucose uptake in muscle
A

B) Glucagon—increased glycogenolysis in skeletal

muscle

How well did you know this?
1
Not at all
2
3
4
5
Perfectly
45
Q
  1. What is a frequent cause of delayed breathing at birth?
    A) Fetal hypoxia during the birth process
    B) Maternal hypoxia during the birth process
    C) Fetal hypercapnia
    D) Maternal hypercapnia
A

A) Fetal hypoxia during the birth process

How well did you know this?
1
Not at all
2
3
4
5
Perfectly
46
Q
55. Which hormone is largely unbound to plasma proteins?
A) Cortisol
B) T4
C) ADH
D) Estradiol
E) Progesterone
A

C) ADH

How well did you know this?
1
Not at all
2
3
4
5
Perfectly
47
Q
  1. What is the mechanism by which the zona pellucida
    becomes “hardened” after penetration of a sperm cell
    to prevent a second sperm from penetrating?
    A) A reduction in estradiol
    B) The proteins released from the acrosome of the
    sperm
    C) An increase in intracellular calcium in the oocyte
    D) An increase in testosterone that affects the sperm
A

C) An increase in intracellular calcium in the oocyte

How well did you know this?
1
Not at all
2
3
4
5
Perfectly
48
Q
  1. Why is milk produced by a woman only after delivery,
    not before?
    A) Levels of LH and FSH are too low during pregnancy to support milk production
    B) High levels of progesterone and estrogen during
    pregnancy suppress milk production
    C) The alveolar cells of the breast do not reach maturity until after delivery
    D) High levels of oxytocin are required for milk production to begin, and oxytocin is not secreted until the baby stimulates the nipple
A

B) High levels of progesterone and estrogen during pregnancy suppress milk production

How well did you know this?
1
Not at all
2
3
4
5
Perfectly
49
Q
  1. Which of the following increases the rate of excretion
    of calcium ions by the kidney?
    A) A decrease in calcitonin concentration in the plasma
    B) An increase in phosphate ion concentration in the
    plasma
    C) A decrease in the plasma level of PTH
    D) Metabolic alkalosis
A

C) A decrease in the plasma level of PTH

How well did you know this?
1
Not at all
2
3
4
5
Perfectly
50
Q
  1. If a woman has a tumor that is secreting large amounts
    of estrogen from the adrenal gland, which of the following will occur?
    A) Progesterone levels in the blood will be very low
    B) Her LH secretion rate will be totally suppressed
    C) She will not have normal menstrual cycles
    D) Her bones will be normally calcified
    E) All the above
A

E) All the above

How well did you know this?
1
Not at all
2
3
4
5
Perfectly
51
Q
  1. Very early in embryonic development, testosterone is
    formed within the male embryo. What is the function
    of this hormone at this stage of development?
    A) Stimulation of bone growth
    B) Stimulation of development of male sex organs
    C) Stimulation of development of skeletal muscle
    D) Inhibition of LH secretion
A

B) Stimulation of development of male sex organs

How well did you know this?
1
Not at all
2
3
4
5
Perfectly
52
Q
  1. Which change would be expected to occur with increased binding of a hormone to plasma proteins?
    A) Increase in plasma clearance of the hormone
    B) Decrease in half-life of the hormone
    C) Increase in hormone activity
    D) Increase in degree of negative feedback exerted by
    the hormone
    E) Increase in plasma reservoir for rapid replenishment of free hormone
A

E) Increase in plasma reservoir for rapid replenishment of free hormone

53
Q

What is the cause of the decrease in calcium ion concentration?
A) The increase in arterial pH resulting from the sodium bicarbonate infusion inhibited PTH secretion
B) The increase in pH resulted in the stimulation of
osteoblasts, which removed calcium from the circulation
C) The increase in pH resulted in an elevation in the
concentration of HPO4−, which shifted the equilibrium between HPO4− and Ca++ toward CaHPO4
D) The increase in arterial pH stimulated the formation of 1,25-dihydroxycholecalciferol, which resulted in an increased rate of absorption of calcium from the gastrointestinal tract

A

C) The increase in pH resulted in an elevation in the

concentration of HPO4−, which shifted the equilibrium between HPO4− and Ca++ toward CaHPO4

54
Q
  1. A patient presents with tachycardia and heat intolerance. You suspect Graves’ disease. Which of the following is not consistent with your diagnosis?
    A) Increased total and free T4
    B) Suppressed plasma [TSH]
    C) Exophthalmos
    D) Goiter
    E) Decreased thyroid radioactive iodine uptake
A

E) Decreased thyroid radioactive iodine uptake

55
Q
  1. A 30-year-old woman is breastfeeding her infant. During suckling, which hormonal response is expected in the woman?
    A) Increased secretion of ADH from the supraoptic nuclei
    B) Increased secretion of ADH from the paraventricular nuclei
    C) Increased secretion of oxytocin from the paraventricular nuclei
    D) Decreased secretion of neurophysin
    E) Increased plasma levels of both oxytocin and ADH
A

C) Increased secretion of oxytocin from the paraventricular nuclei

56
Q
  1. Why is it important to feed newborn infants every few
    hours?
    A) The hepatic capacity to store and synthesize glycogen and glucose is not adequate to maintain the plasma glucose concentration in a normal range for more than a few hours after feeding
    B) If adequate fluid is not ingested frequently, the plasma protein concentration will rise to greater than normal levels within a few hours
    C) The function of the gastrointestinal system is poorly developed and can be improved by keeping food in the stomach at all times
    D) The hepatic capacity to form plasma proteins is minimal and requires the constant availability of amino acids from food to avoid hypoproteinemic edema
A

A) The hepatic capacity to store and synthesize glycogen and glucose is not adequate to maintain the plasma glucose concentration in a normal range for more than a few hours after feeding

57
Q
70. Dehydroepiandrosterone sulfate (DHEAS), the precursor for the high levels of estradiol that occur in pregnancy, is made in what tissue?
A) Fetal adrenal gland
B) Ovary of the mother
C) Placenta
D) Adrenal gland of the mother
A

D) Adrenal gland of the mother

58
Q
  1. What is the consequence of sporadic nursing of the
    neonate by the mother?
    A) An increase in prolactin-releasing hormone
    B) An increase in oxytocin
    C) Lack of birth control
    D) Lack of prolactin surge
A

D) Lack of prolactin surge

59
Q
  1. Which of the following would be associated with parallel changes in aldosterone and cortisol secretion?
    A) Addison’s disease
    B) Cushing’s disease
    C) Cushing’s syndrome (adrenal tumor)
    D) A low-sodium diet
    E) Administration of a converting enzyme inhibitor
A

A) Addison’s disease

60
Q
  1. A chronic increase in the plasma concentration of
    thyroxine-binding globulin (TBG) would result in
    which of the following?
    A) An increased delivery of T4 to target cells
    B) A decrease in plasma free [T4]
    C) An increase in the conversion of T4 to triiodothyronine (T3) in peripheral tissues
    D) An increase in TSH secretion
    E) No change in metabolic rate
A

E) No change in metabolic rate

61
Q
  1. RU486 causes abortion if it is administered before or
    soon after implantation. What is the specific effect of
    RU486?
    A) It binds to LH receptors, stimulating the secretion
    of progesterone from the corpus luteum
    B) It blocks progesterone receptors so that progesterone has no effect within the body
    C) It blocks the secretion of FSH by the pituitary
    D) It blocks the effects of oxytocin receptors in the
    uterine muscle
A

B) It blocks progesterone receptors so that progesterone has no effect within the body

62
Q
  1. A 55-year-old man has developed the syndrome of
    inappropriate antidiuretic hormone secretion due to
    carcinoma of the lung. Which physiological response
    would be expected?
    A) Increased plasma osmolality
    B) Inappropriately low urine osmolality (relative to
    plasma osmolality)
    C) Increased thirst
    D) Decreased secretion of ADH from the pituitary
    gland
A

D) Decreased secretion of ADH from the pituitary

gland

63
Q
  1. During pregnancy, the uterine smooth muscle is quiescent. During the ninth month of gestation, the uterine muscle becomes progressively more excitable. What factor contributes to the increase in excitability?
    A) Placental estrogen synthesis rises to high rates
    B) Progesterone synthesis by the placenta decreases
    C) Uterine blood flow reaches its highest rate
    D) PGE2 synthesis by the placenta decreases
    E) Activity of the fetus falls to low levels
A

B) Progesterone synthesis by the placenta decreases

64
Q
  1. A 20-year-old woman is not having menstrual cycles.
    Her plasma progesterone concentration is found to be
    minimal. What is the explanation for the low level of
    progesterone?
    A) LH secretion rate is elevated
    B) LH secretion rate is suppressed
    C) FSH secretion rate is suppressed
    D) No corpus luteum is present
    E) High inhibin concentration in the plasma has suppressed progesterone synthesis
A

D) No corpus luteum is present

65
Q
78. Before the preovulatory surge in LH, granulosa cells of 
the follicle secrete which hormone?
A) Testosterone
B) Progesterone
C) Estrogen
D) Inhibin
A

C) Estrogen

66
Q
81. Which hormone activates enzyme-linked receptors?
A) ADH
B) Insulin
C) ACTH
D) PTH
E) Aldosterone
A

B) Insulin

67
Q
  1. Which of the following is produced by the trophoblast
    cells during the first 3 weeks of pregnancy?
A) Estrogen
B) LH
C) Oxytocin
D) HCG
E) None of the above
A

D) HCG

68
Q
  1. Which of the following is higher in the neonate than in
    the fetus?

A) Flow through the foramen ovale
B) Right atrial pressure
C) Flow through the ductus arteriosus
D) Aortic pressure

A

B) Right atrial pressure

69
Q
  1. Which finding is most likely in a patient who has myxedema?
A) Somnolence
B) Palpitations
C) Increased respiratory rate
D) Increased cardiac output
E) Weight loss
A

A) Somnolence

70
Q
  1. At birth, a large, well-nourished baby is found to have
    a plasma glucose concentration of 17 mg/dl (normal is
    80 to 100 mg/dl) and a plasma insulin concentration
    twice the normal value. What is the explanation for
    these findings?
    A) The neonate experienced in utero malnutrition
    B) The mother was malnourished during pregnancy
    C) The mother is diabetic, with poorly controlled
    hyperglycemia
    D) The mother is obese
A

C) The mother is diabetic, with poorly controlled

hyperglycemia

71
Q
  1. In the fetus, why can normal growth occur despite low
    Po2 levels?
    A) The concentration of hemoglobin A is increased in
    the fetus
    B) The hemoglobin of the fetus can carry more oxygen
    at lower Pco2 levels
    C) The oxyhemoglobin curve in the fetus is shifted to
    the left
    D) The mother has increased blood volume during
    pregnancy
A

C) The oxyhemoglobin curve in the fetus is shifted to

the left

72
Q
  1. Which of the following stimulates the secretion of
    PTH?
    A) An increase in extracellular calcium ion activity
    above the normal value
    B) An increase in calcitonin concentration
    C) Respiratory acidosis
    D) Increased secretion of PTH-releasing hormone
    from the hypothalamus
    E) None of the above
A

E) None of the above

73
Q
88. A 40-year-old woman consumes a high-potassium diet 
for several weeks. Which hormonal change is most 
likely to occur?
A) Increased secretion of DHEA
B) Increased secretion of cortisol
C) Increased secretion of aldosterone
D) Increased secretion of ACTH
E) Decreased secretion of CRH
A

C) Increased secretion of aldosterone

74
Q
89. After implantation into the uterus, nutrition of the 
blastocyst comes from which structure?
A) Placenta
B) Decidua
C) Glomerulosa cells
D) Corpus luteum
A

B) Decidua

75
Q
90. Which hormone is not stored in its endocrine-producing 
gland?
A) T4
B) PTH
C) Aldosterone
D) ACTH
E) Insulin
A

C) Aldosterone

76
Q
  1. A young woman comes to the emergency department
    with a vertebral compression fracture. Radiographs of
    the spine indicate generalized demineralization. She is
    vegetarian, does not smoke or drink alcohol, and has a
    normal plasma potassium concentration of 5.4 mEq/L,
    a sodium concentration of 136 mEq/L, and a plasma
    calcium concentration of 7.0 mg/dl. Her vitamin D3
    value is several times greater than normal, although her
    1,25-dihydroxycholecalciferol concentration is at the
    lower limit of detectability. She has been in renal failure
    for the past 5 years and undergoes hemodialysis three
    times each week. What is the cause of her low 1,25-dihydroxycholecalciferol level?
    A) Metabolic acidosis
    B) Metabolic alkalosis
    C) She is unable to form 1,25-dihydroxycholecalciferol
    because of her extensive kidney disease
    D) She is undergoing dialysis with a dialysis fluid that
    does not contain calcium
    E) She is taking receiving calcium supplements
A

C) She is unable to form 1,25-dihydroxycholecalciferol

because of her extensive kidney disease

77
Q
92. The placenta is incapable of synthesizing which hormones?
A) Estrogen
B) Progesterone
C) Androgens
D) Estriol
A

C) Androgens

78
Q
  1. A neonate develops jaundice and has a bilirubin concentration of 10 mg/dl on day 2 (normal is 3 mg/dl at 2
    days old). The neonatologist can be confident that the
    condition is not erythroblastosis fetalis if which of the
    following is true?
    A) The bilirubin concentration rises no further
    B) Hematocrit falls only slightly
    C) The mother, father, and neonate are all Rh-negative
    D) The mother has no history of hepatic dysfunction
A

C) The mother, father, and neonate are all Rh-negative

79
Q
94. Which finding would likely be reported in a patient 
with a deficiency in iodine intake?
A) Weight loss
B) Nervousness
C) Increased sweating
D) Increased synthesis of thyroglobulin
E) Tachycardia
A

D) Increased synthesis of thyroglobulin

80
Q
  1. A 37-year-old woman presents to her physician with an
    enlarged thyroid gland and high plasma levels of T4 and
    T3. Which of the following is likely to be decreased?
    A) Heart rate
    B) Cardiac output
    C) Peripheral vascular resistance
    D) Ventilation rate
    E) Metabolic rate
A

C) Peripheral vascular resistance

81
Q
  1. Before intercourse, a woman irrigates her vagina with
    a solution that lowers the pH of the vaginal fluid to 4.5.
    What will be the effect on sperm cells in the vagina?
    A) The metabolic rate will increase
    B) The rate of movement will decrease
    C) The formation of PGE2 will increase
    D) The rate of oxygen consumption will increase
A

B) The rate of movement will decrease

82
Q
  1. Men who take large doses of testosterone-like androgenic steroids for long periods are sterile in the reproductive sense of the word. What is the explanation for this finding?
    A) High levels of androgens bind to testosterone receptors in the Sertoli cells, resulting in overstimulation of inhibin formation
    B) Overstimulation of sperm cell production results
    in the formation of defective sperm cells
    C) High levels of androgen compounds inhibit the
    secretion of GnRH by the hypothalamus, resulting in the inhibition of LH and FSH release by the anterior pituitary
    D) High levels of androgen compounds produce hypertrophic dysfunction of the prostate gland
A

C) High levels of androgen compounds inhibit the

secretion of GnRH by the hypothalamus, resulting in the inhibition of LH and FSH release by the anterior pituitary

83
Q
99. Cortisone is administered to a 30-year-old woman for 
the treatment of an autoimmune disease. Which of 
the following is most likely to occur?
A) Increased ACTH secretion
B) Increased cortisol secretion
C) Increased insulin secretion
D) Increased muscle mass
E) Hypoglycemia between meals
A

C) Increased insulin secretion

84
Q
  1. In the hypothalamic-pituitary-gonadal axis of the
    female, what is the follicular cell type that produces
    inhibin?
    A) Cytotrophoblasts
    B) Synthiotrophoblasts
    C) Granulosa
    D) Thecal
A

C) Granulosa

85
Q
  1. The function of which of the following is increased by
    an elevated parathyroid hormone concentration?
    A) Osteoclasts
    B) Hepatic formation of 25-hydroxycholecalciferol
    C) Phosphate reabsorptive pathways in the renal tubules
    D) All the above
A

A) Osteoclasts

86
Q
  1. Which statement about peptide or protein hormones
    is usually true?
    A) They have longer half-lives than steroid hormones
    B) They have receptors on the cell membrane
    C) They have a slower onset of action than both steroid and thyroid hormones
    D) They are not stored in endocrine-producing glands
A

B) They have receptors on the cell membrane

87
Q
104. Cortisol and GH are most dissimilar in their metabolic effects on which of the following?
A) Protein synthesis in muscle
B) Glucose uptake in peripheral tissues
C) Plasma glucose concentration
D) Mobilization of triglycerides
A

A) Protein synthesis in muscle

88
Q
  1. Why do infants of mothers who had adequate nutrition
    during pregnancy not require iron supplements or a
    diet rich in iron until about 3 months of age?
    A) Growth of the infant does not require iron until
    after the third month
    B) The fetal liver stores enough iron to meet the infant’s needs until the third month
    C) Synthesis of new red blood cells begins after 3
    months
    D) Muscle cells that develop before the third month
    do not contain myoglobin
A

B) The fetal liver stores enough iron to meet the infant’s needs until the third month

89
Q
106. Cortisone is administered to a patient for the treatment of an autoimmune disease. Which of the following would least likely occur in response to the cortisone 
treatment?
A) Hypertrophy of the adrenal glands
B) Increased plasma levels of C-peptide
C) Decreased CRH secretion
D) Increased blood pressure
E) Hyperglycemia
A

A) Hypertrophy of the adrenal glands

90
Q
107. Which symptom would least likely be associated with 
thyrotoxicosis?
A) Tachycardia
B) Increased appetite
C) Somnolence
D) Increased sweating
E) Muscle tremor
A

C) Somnolence

91
Q
  1. If a male is born without a penis and testes, a defect is
    likely in which gene on the Y chromosome?
    A) ERE—estrogen response element
    B) ARE—androgen response element
    C) SRY—affecting Sertoli cells
    D) ERG—early response genes
A

C) SRY—affecting Sertoli cells

92
Q
109. Where does fertilization normally take place?
A) Uterus
B) Cervix
C) Ovary
D) Ampulla of the fallopian tubes
A

D) Ampulla of the fallopian tubes

93
Q
  1. Which finding is most likely to occur in a patient who
    has uncontrolled type 1 DM?
    A) Decreased plasma osmolality
    B) Increased plasma volume
    C) Increased plasma pH
    D) Increased release of glucose from the liver
    E) Decreased rate of lipolysis
A

D) Increased release of glucose from the liver

94
Q
111. GH secretion would most likely be suppressed under 
which condition?
A) Acromegaly
B) Gigantism
C) Deep sleep
D) Exercise
E) Acute hyperglycemia
A

E) Acute hyperglycemia

95
Q
112. Pregnenolone is not in the biosynthetic pathway of 
which substance?
A) Cortisol
B) Estrogen
C) Aldosterone
D) 1,25(OH)2D
E) DHEA
A

D) 1,25(OH)2D

96
Q
  1. Two days before the onset of menstruation, secretions
    of FSH and LH reach their lowest levels. What is the
    cause of this low level of secretion?
    A) The anterior pituitary gland becomes unresponsive to the stimulatory effect of GnRH
    B) Estrogen from the developing follicles exerts a
    feedback inhibition on the hypothalamus
    C) The rise in body temperature inhibits hypothalamic release of GnRH
    D) Secretion of estrogen, progesterone, and inhibin
    by the corpus luteum suppresses hypothalamic
    secretion of GnRH and pituitary secretion of FSH
A

D) Secretion of estrogen, progesterone, and inhibin
by the corpus luteum suppresses hypothalamic
secretion of GnRH and pituitary secretion of FSH

97
Q
  1. Which condition contributes to “sodium escape” in
    persons with Conn’s syndrome?

A) Decreased plasma levels of atrial natriuretic peptide
B) Increased plasma levels of angiotensin II
C) Decreased sodium reabsorption in the collecting
tubules
D) Increased arterial pressure

A

D) Increased arterial pressure

98
Q
  1. A 30-year-old woman reports to the clinic for a routine physical examination, which reveals she is pregnant. Her plasma levels of TSH are high, but her total
    T4 concentration (protein bound and free) is normal.
    Which of the following best reflects this patient’s clinical state?
    A) Graves’ disease
    B) Hashimoto’s disease
    C) A pituitary tumor that is secreting TSH
    D) A hypothalamic tumor that is secreting TRH
    E) The patient is taking thyroid extract
A

B) Hashimoto’s disease

99
Q
  1. A man has a disease that destroyed only the motor
    neurons of the spinal cord below the thoracic region. Which aspect of sexual function would not be
    possible?
    A) Arousal
    B) Erection
    C) Lubrication
    D) Ejaculation
A

D) Ejaculation

100
Q
  1. Which component of the reproductive system has the
    most far-reaching effects on the physiology of the organism?
    A) /Y chromosomal effects
    B) X dose—one X chromosome versus two X chromosomes
    C) Gonadal steroid hormones
    D) Prenatal testosterone levels
A

C) Gonadal steroid hormones

101
Q
  1. A sustained program of lifting heavy weights will increase bone mass. What is the mechanism of this
    effect of weightlifting?
    A) Elevated metabolic activity stimulates parathyroid
    hormone secretion
    B) Mechanical stress on the bones increases the activity of osteoblasts
    C) Elevated metabolic activity results in an increase
    in dietary calcium intake
    D) Elevated metabolic activity results in stimulation
    of calcitonin secretion
A

B) Mechanical stress on the bones increases the activity of osteoblasts

102
Q
  1. Birth control pills containing combinations of synthetic estrogen and progesterone compounds that are
    given for the first 21 days of the menstrual cycle are
    effective in preventing pregnancy. What is the explanation for their efficacy?
    A) Prevention of the preovulatory surge of LH secretion from the pituitary gland
    B) Prevention of development of the ovarian follicles
    C) Suppression of the function of the corpus luteum
    soon after it forms
    D) Prevention of normal development of the endometriu
A

A) Prevention of the preovulatory surge of LH secretion from the pituitary gland

103
Q
  1. Which of the following would be expected in a patient
    with a genetic deficiency of 11-β-hydroxysteroid dehydrogenase type II?
    A) Hyperkalemia
    B) Hypertension
    C) Increased plasma renin activity
    D) Increased plasma [aldosterone]
    E) Hyperglycemia
A

B) Hypertension

104
Q
  1. Which physiological response is greater for T3 than
    for T4?
    A) Secretion rate from the thyroid
    B) Plasma concentration
    C) Plasma half-life
    D) Affinity for nuclear receptors in target tissues
    E) Latent period for the onset of action in target
    tissues
A

D) Affinity for nuclear receptors in target tissues

105
Q
  1. A “birth control” compound for men has been sought
    for several decades. Which substance would provide
    effective sterility?
    A) A substance that mimics the actions of LH
    B) A substance that blocks the actions of inhibin
    C) A substance that blocks the actions of FSH
    D) A substance that mimics the actions of GnRH
A

C) A substance that blocks the actions of FSH

106
Q
  1. For milk to flow from the nipple of the mother into
    the mouth of the nursing infant, what must occur?
    A) Myoepithelial cells must relax
    B) Prolactin levels must fall
    C) Oxytocin secretion from the posterior pituitary
    must take place
    D) The baby’s mouth must develop a strong negative
    pressure over the nipple
    E) All the above
A

C) Oxytocin secretion from the posterior pituitary

must take place

107
Q
  1. Failure of the ductus arteriosus to close is a common
    developmental defect. Which condition would likely
    be present in a 12-month-old infant with patent ductus arteriosus?
    A) Below normal arterial Po2
    B) Below normal arterial Pco2
    C) Greater than normal arterial blood pressure
    D) Lower than normal pulmonary arterial pressure
A

A) Below normal arterial Po2

108
Q
  1. When compared with the late-evening values typically observed in normal subjects, plasma levels of both ACTH and cortisol would be expected to be higher in which persons?

A) Normal subjects after waking in the morning
B) Normal subjects who have taken dexamethasone
C) Patients with Cushing’s syndrome (adrenal adenoma)
D) Patients with Addison’s disease
E) Patients with Conn’s syndrome

A

A) Normal subjects after waking in the morning

109
Q
128. Which of the following conditions or hormones 
would most likely increase GH secretion?
A) Hyperglycemia
B) Exercise
C) Somatomedin
D) Somatostatin
E) Aging
A

B) Exercise

110
Q
  1. What would be associated with parallel changes in aldosterone and cortisol secretion?
    A) Addison’s disease
    B) Cushing’s disease
    C) Cushing’s syndrome (ectopic ACTH-producing
    tumor)
    D) A high-sodium diet
    E) Administration of a converting enzyme inhibitor
A

A) Addison’s disease

111
Q
131. Which blood vessel in the fetus has the highest Po2?
A) Ductus arteriosus
B) Ductus venosus
C) Ascending aorta
D) Left atrium
A

B) Ductus venosus

112
Q
  1. A 59-year-old woman has osteoporosis, hypertension, hirsutism, and hyperpigmentation. Magnetic resonance imaging indicates that the pituitary gland is not enlarged. Which condition is most consistent with these findings?
A) Pituitary ACTH-secreting tumor
B) Ectopic ACTH-secreting tumor
C) Inappropriately high secretion rate of CRH
D) Adrenal adenoma
E) Addison’s disease
A

B) Ectopic ACTH-secreting tumor

113
Q
  1. During the latter stages of pregnancy, many women
    experience an increase in body hair growth in a masculine pattern. What is the explanation for this phenomenon?
    A) The ovaries secrete some testosterone along with
    the large amounts of estrogen produced late in
    pregnancy
    B) The fetal ovaries and testes secrete androgenic
    steroids
    C) The maternal and fetal adrenal glands secrete
    large amounts of androgenic steroids that are used
    by the placenta to form estrogen
    D) The placenta secretes large amounts of estrogen,
    some of which is metabolized to testosterone
A

C) The maternal and fetal adrenal glands secrete
large amounts of androgenic steroids that are used
by the placenta to form estrogen

114
Q
  1. What causes menopause?
    A) Reduced levels of gonadotropic hormones secreted
    from the anterior pituitary gland
    B) Reduced responsiveness of the follicles to the
    stimulatory effects of gonadotropic hormones
    C) Reduced rate of secretion of progesterone from
    the corpus luteum
    D) Reduced numbers of follicles available in the ovary for stimulation by gonadotropic hormones
A

D) Reduced numbers of follicles available in the ovary for stimulation by gonadotropic hormones

115
Q
  1. What does not increase when insulin binds to its
    receptor?
    A) Fat synthesis in adipose tissue
    B) Protein synthesis in muscle
    C) Glycogen synthesis
    D) Gluconeogenesis in the liver
    E) Intracellular tyrosine kinase activity
A

D) Gluconeogenesis in the liver

116
Q
138. Release of which hormone is an example of neuroendocrine secretion?
A) GH
B) Cortisol
C) Oxytocin
D) Prolactin
E) ACTH
A

C) Oxytocin

117
Q
  1. During the week after ovulation, the endometrium
    increases in thickness to 5 to 6 millimeters. What
    stimulates this increase in thickness?
    A) LH
    B) Estrogen from the corpus luteum
    C) Progesterone from the corpus luteum
    D) FSH
A

C) Progesterone from the corpus luteum

118
Q
140. Inhibition of the iodide pump would be expected to 
cause which change?
A) Increased synthesis of T4
B) Increased synthesis of thyroglobulin
C) Increased metabolic rate
D) Decreased TSH secretion
E) Extreme nervousness
A

B) Increased synthesis of thyroglobulin

119
Q
  1. Before implantation, the blastocyst obtains its nutrition from uterine endometrial secretions. How does
    the blastocyst obtain nutrition during the first week
    after implantation?
    A) It continues to derive nutrition from endometrial
    secretions
    B) The cells of the blastocyst contain stored nutrients
    that are metabolized for nutritional support
    C) The placenta provides nutrition derived from
    maternal blood
    D) The trophoblast cells digest the nutrient-rich endometrial cells and then absorb their contents for
    use by the blastocyst
A

D) The trophoblast cells digest the nutrient-rich endometrial cells and then absorb their contents for
use by the blastocyst

120
Q
142. Which pituitary hormone has a chemical structure 
most similar to that of ADH?
A) Oxytocin
B) ACTH
C) TSH
D) FSH
E) Prolactin
A

A) Oxytocin

121
Q
143. Which option would not be efficacious in the treatment of patients with type 2 diabetes?
A) Glucocorticoids
B) Insulin injections
C) Thiazolidinediones
D) Sulfonylureas
E) Weight loss
A

A) Glucocorticoids

122
Q
  1. Which of the following is most likely to occur in the
    early stages of type 2 diabetes?
    A) Increased insulin sensitivity
    B) Decreased hepatic glucose output
    C) Increased plasma levels of C-peptide
    D) Increased plasma [β-hydroxybutyric acid]
    E) Hypovolemia
A

C) Increased plasma levels of C-peptide

123
Q
  1. What is the most common cause of respiratory distress syndrome in neonates born at 7 months’ gestation?
    A) Pulmonary edema due to pulmonary arterial
    hypertension
    B) Formation of a hyaline membrane over the alveolar surface
    C) Failure of the alveolar lining to form adequate
    amounts of surfactant
    D) Excessive permeability of the alveolar membrane
    to water
A

C) Failure of the alveolar lining to form adequate

amounts of surfactant

124
Q
  1. A 45-year-old woman has a mass in the sella turcica
    that compresses the portal vessels, disrupting pituitary access to hypothalamic secretions. The secretion
    rate of which hormone would most likely increase in
    this patient?
    A) ACTH
    B) GH
    C) Prolactin
    D) LH
    E) TSH
A

C) Prolactin

125
Q
148. Which of the following is not produced by osteoblasts?
A) Alkaline phosphatase
B) RANK ligand
C) Collagen
D) Pyrophosphate
E) Osteoprotegerin
A

D) Pyrophosphate

126
Q
  1. A man who has been exposed to high levels of gamma
    radiation is sterile due to destruction of the germinal
    epithelium of the seminiferous tubules, although he
    has normal levels of testosterone. Which of the following would be found in this patient?
    A) A normal secretory pattern of GnRH
    B) Normal levels of inhibin
    C) Suppressed levels of FSH
    D) Absence of Leydig cells
A

A) A normal secretory pattern of GnRH

127
Q
151. What is hormone 1?
A) TRH
B) TSH
C) T4
D) ACTH
E) Cortisol
A

B) TSH

128
Q
152. What is hormone 2?
A) TSH
B) T4
C) CRH
D) ACTH
E) Cortisol
A

C) CRH